Is there an easier way to solve these basic integration problems?

  • Thread starter Thread starter ming2194
  • Start date Start date
Click For Summary
The discussion revolves around solving basic integration problems, with participants seeking guidance on specific integrals. One user questions if they can simplify expressions like e^2x and whether their approach to integration is correct. Responses indicate that while some methods may work, others suggest using improper integrals due to undefined points in the integrand. There is also a focus on verifying the correctness of solutions by differentiating results. The conversation emphasizes the importance of recognizing when to apply improper integral techniques and suggests alternative methods for solving complex integrals.
ming2194
Messages
31
Reaction score
0

Homework Statement


http://u1.imgupload.co.uk/1257897600/5a0e_image.bmp

Homework Equations


above

The Attempt at a Solution


1)
needs some hints here.
may i treat e^2x as (e^x)2 and then apply the formula to get the answers?

2)
http://u1.imgupload.co.uk/1257897600/bd3b_1.png

i didnt write a full attempt but it is the flow of my thought.
did my attempt correct? or in fact there are other easier way to get the answers?

3)
http://u1.imgupload.co.uk/1257897600/96d0_2.bmp
this one i just wonder whether my work is correct or not. (shown in pic)

thx a lots.:blushing:
 
Physics news on Phys.org
1) Assuming the formula is for the standard form \int \frac{1}{\sqrt{x^2-1}} dx, no you can not treat it that simply. You could try letting u=e^x.

2) Your method will work. I've tried some things and that seems to be the most elementary.

3) That is pretty much the correct evaluation of the "net area" but strictly the integral does not converge at all because the integrand is not well defined over the interval of integration.
 
For 3, as Gib Z already noted, the integrand is undefined at a point in the interval. For this problem you need to split the integral into two improper integrals:
\lim_{a \rightarrow 1^-} \int_0^a \frac{dx}{x - 1}~+~\lim_{b \rightarrow 1^+} \int_b^3 \frac{dx}{x - 1}
Evaluate each integral and take the limit. There's no guarantee that either improper integral will converge, though.
 
actually how can i know when the question is asking me using improper integral? or it's needed to decide by myself?

and for 2), i worked out the answer and found it's damn long and complex. so how can i check whether I am correct?

1)
http://u1.imgupload.co.uk/1257897600/492b_aaa.png
please judge

thx again.
 
I think this is wrong, which you can confirm by differentiating your final result. How did you go from this integral:
\int \frac{du}{\sqrt{u^4 - 4u^2}}
to this?
ln(u^2 + \sqrt{u^4 - 4u^2}) + C

I would have left the integral like this:
\int \frac{du}{u\sqrt{u^2 - 4}}
and worked toward a trig substitution, with sec(\theta) = u/2.
 
Question: A clock's minute hand has length 4 and its hour hand has length 3. What is the distance between the tips at the moment when it is increasing most rapidly?(Putnam Exam Question) Answer: Making assumption that both the hands moves at constant angular velocities, the answer is ## \sqrt{7} .## But don't you think this assumption is somewhat doubtful and wrong?

Similar threads

  • · Replies 5 ·
Replies
5
Views
2K
  • · Replies 4 ·
Replies
4
Views
1K
  • · Replies 2 ·
Replies
2
Views
1K
Replies
5
Views
2K
  • · Replies 13 ·
Replies
13
Views
1K
Replies
2
Views
3K
  • · Replies 21 ·
Replies
21
Views
2K
Replies
7
Views
2K
  • · Replies 1 ·
Replies
1
Views
2K
  • · Replies 4 ·
Replies
4
Views
2K